Let \(W\) be the subspace spanned by the vectors \[ \begin{bmatrix} 1 \\ -3 \\ -10 \\ 2 \end{bmatrix}, \qquad \begin{bmatrix} 4 \\ 4 \\ -8 \\ -24 \end{bmatrix}.\] Find a basis for the orthogonal complement \(W^\perp\) of \(W\).
Click for answer
Click for Latex code
Let \(W\) be the subspace spanned by the vectors
\[
\begin{bmatrix}
1 \\
-3 \\
-10 \\
2
\end{bmatrix}, \qquad
\begin{bmatrix}
4 \\
4 \\
-8 \\
-24
\end{bmatrix}.\]
Find a basis for the orthogonal complement \(W^\perp\) of \(W\).
\begin{solution}
Solution:
Let \(A\) be the matrix with columns the above vectors:
\[\begin{bmatrix}
1 & 4 \\
-3 & 4 \\
-10 & -8 \\
2 & -24
\end{bmatrix}\]
Reduced row echelon form (RREF) of \(A^T\):
\[\begin{bmatrix}
1 & 0 & -4 & -4 \\
0 & 1 & 2 & -2
\end{bmatrix}\]
Basis for \(W^\perp = (\textrm{Col}A)^\perp = \textrm{Nul} A^T\):
\[\begin{bmatrix}
4 \\
-2 \\
1 \\
0
\end{bmatrix}, \qquad
\begin{bmatrix}
4 \\
2 \\
0 \\
1
\end{bmatrix}.\]
\end{solution}